Evaluate $sum_{r=2}^{infty} frac{2-r}{r(r+1)(r+2)}$












5












$begingroup$



Evaluate
$$sum_{r=2}^{infty} frac{2-r}{r(r+1)(r+2)}$$




So in a previous part of the question I calculated that
$$sum_{r=1}^{n} frac{2-r}{r(r+1)(r+2)} = sum_{r=1}^{n}left( frac{1}{r}-frac{3}{r+1}+frac{2}{r+2}right)=frac{n}{(n+1)(n+2)}$$



So my question is then why does $$sum_{r=2}^{infty} frac{2-r}{r(r+1)(r+2)} = -frac{1}{6}$$



As I though that it would be $frac{1}{2}$.










share|cite|improve this question









$endgroup$

















    5












    $begingroup$



    Evaluate
    $$sum_{r=2}^{infty} frac{2-r}{r(r+1)(r+2)}$$




    So in a previous part of the question I calculated that
    $$sum_{r=1}^{n} frac{2-r}{r(r+1)(r+2)} = sum_{r=1}^{n}left( frac{1}{r}-frac{3}{r+1}+frac{2}{r+2}right)=frac{n}{(n+1)(n+2)}$$



    So my question is then why does $$sum_{r=2}^{infty} frac{2-r}{r(r+1)(r+2)} = -frac{1}{6}$$



    As I though that it would be $frac{1}{2}$.










    share|cite|improve this question









    $endgroup$















      5












      5








      5





      $begingroup$



      Evaluate
      $$sum_{r=2}^{infty} frac{2-r}{r(r+1)(r+2)}$$




      So in a previous part of the question I calculated that
      $$sum_{r=1}^{n} frac{2-r}{r(r+1)(r+2)} = sum_{r=1}^{n}left( frac{1}{r}-frac{3}{r+1}+frac{2}{r+2}right)=frac{n}{(n+1)(n+2)}$$



      So my question is then why does $$sum_{r=2}^{infty} frac{2-r}{r(r+1)(r+2)} = -frac{1}{6}$$



      As I though that it would be $frac{1}{2}$.










      share|cite|improve this question









      $endgroup$





      Evaluate
      $$sum_{r=2}^{infty} frac{2-r}{r(r+1)(r+2)}$$




      So in a previous part of the question I calculated that
      $$sum_{r=1}^{n} frac{2-r}{r(r+1)(r+2)} = sum_{r=1}^{n}left( frac{1}{r}-frac{3}{r+1}+frac{2}{r+2}right)=frac{n}{(n+1)(n+2)}$$



      So my question is then why does $$sum_{r=2}^{infty} frac{2-r}{r(r+1)(r+2)} = -frac{1}{6}$$



      As I though that it would be $frac{1}{2}$.







      summation






      share|cite|improve this question













      share|cite|improve this question











      share|cite|improve this question




      share|cite|improve this question










      asked Dec 31 '18 at 15:47









      H.LinkhornH.Linkhorn

      537313




      537313






















          6 Answers
          6






          active

          oldest

          votes


















          7












          $begingroup$

          Note thatbegin{align}frac{2-r}{r(r+1)(r+2)}&=frac1r-frac3{r+1}+frac2{r+2}\&=frac1r-frac1{r+1}-frac2{r+1}+frac2{r+2}.end{align}Therefore, your series is naturally the sum of two telescoping series and its sum is$$frac12-frac23=-frac16.$$






          share|cite|improve this answer









          $endgroup$













          • $begingroup$
            (+1) was a minute late!
            $endgroup$
            – TheSimpliFire
            Dec 31 '18 at 15:54



















          5












          $begingroup$

          Note that begin{align}sum_{r=2}^inftyleft( frac{1}{r}-frac{3}{r+1}+frac{2}{r+2}right)&=sum_{r=2}^inftyleft(frac1r-frac1{r+1}right)-2sum_{r=2}^inftyleft(frac1{r+1}-frac1{r+2}right)\&=left(frac12-frac13+frac13-frac14+cdotsright)-2left(frac13+frac14-frac14+frac15-cdotsright)\&=frac12-frac23=-frac16end{align}






          share|cite|improve this answer









          $endgroup$









          • 1




            $begingroup$
            (+1) Believe me: I know what it feels to be a few seconds too late.
            $endgroup$
            – José Carlos Santos
            Dec 31 '18 at 15:55



















          1












          $begingroup$

          Actually your finite sum is equal to $$sum_{r=2}^nfrac{2-r}{r(r+1)(r+2)}=frac{2}{n+2}-frac{1}{n+1}-frac{1}{6}$$






          share|cite|improve this answer









          $endgroup$





















            1












            $begingroup$

            Let $dfrac{2-r}{r(r+1)(r+2)}=f(r)-f(r+1),$ where$ f(m)=dfrac{am+b}{m(m+1)}$



            $implies2-r=(r+2)(ar+b)-r(ar+a+b)=ar+2b$



            Set $2b=2iff b=1,ar=-riff a=-1$



            $$impliessum_{r=1}^ndfrac{2-r}{r(r+1)(r+2)}=sum_{r=1}^nleft(f(r)-f(r+1)right)=f(1)-f(n+1)$$



            Set $ntoinfty$ and show that $ntoinfty f(n)=0$






            share|cite|improve this answer









            $endgroup$





















              1












              $begingroup$

              If $H_{n}$ is the $n$-th harmonic number, we have $$sum_{r=2}^{n}left(frac{1}{r}-frac{3}{r+1}+frac{2}{r+2}right)=left(H_{n}-1right)-3left(H_{n}-1-frac{1}{2}right)+2left(H_{n}-1-frac{1}{2}-frac{1}{3}right)$$ $$=-1+frac{3}{2}-frac{2}{3}=-frac{1}{6}.$$ Now take the limit as $nrightarrow+infty$.






              share|cite|improve this answer









              $endgroup$





















                0












                $begingroup$

                By definition,



                $$ sum_{r=1}^infty a_r = lim_{ntoinfty}sum_{r=1}^n a_r .$$



                In your case the above limit is zero. Then:



                $$ sum_{r=2}^infty a_r = sum_{r=1}^infty a_r - a_1 = -a_1 . $$



                This reasoning is what @HenriLee was referring to in his answer.






                share|cite|improve this answer









                $endgroup$














                  Your Answer








                  StackExchange.ready(function() {
                  var channelOptions = {
                  tags: "".split(" "),
                  id: "69"
                  };
                  initTagRenderer("".split(" "), "".split(" "), channelOptions);

                  StackExchange.using("externalEditor", function() {
                  // Have to fire editor after snippets, if snippets enabled
                  if (StackExchange.settings.snippets.snippetsEnabled) {
                  StackExchange.using("snippets", function() {
                  createEditor();
                  });
                  }
                  else {
                  createEditor();
                  }
                  });

                  function createEditor() {
                  StackExchange.prepareEditor({
                  heartbeatType: 'answer',
                  autoActivateHeartbeat: false,
                  convertImagesToLinks: true,
                  noModals: true,
                  showLowRepImageUploadWarning: true,
                  reputationToPostImages: 10,
                  bindNavPrevention: true,
                  postfix: "",
                  imageUploader: {
                  brandingHtml: "Powered by u003ca class="icon-imgur-white" href="https://imgur.com/"u003eu003c/au003e",
                  contentPolicyHtml: "User contributions licensed under u003ca href="https://creativecommons.org/licenses/by-sa/3.0/"u003ecc by-sa 3.0 with attribution requiredu003c/au003e u003ca href="https://stackoverflow.com/legal/content-policy"u003e(content policy)u003c/au003e",
                  allowUrls: true
                  },
                  noCode: true, onDemand: true,
                  discardSelector: ".discard-answer"
                  ,immediatelyShowMarkdownHelp:true
                  });


                  }
                  });














                  draft saved

                  draft discarded


















                  StackExchange.ready(
                  function () {
                  StackExchange.openid.initPostLogin('.new-post-login', 'https%3a%2f%2fmath.stackexchange.com%2fquestions%2f3057821%2fevaluate-sum-r-2-infty-frac2-rrr1r2%23new-answer', 'question_page');
                  }
                  );

                  Post as a guest















                  Required, but never shown

























                  6 Answers
                  6






                  active

                  oldest

                  votes








                  6 Answers
                  6






                  active

                  oldest

                  votes









                  active

                  oldest

                  votes






                  active

                  oldest

                  votes









                  7












                  $begingroup$

                  Note thatbegin{align}frac{2-r}{r(r+1)(r+2)}&=frac1r-frac3{r+1}+frac2{r+2}\&=frac1r-frac1{r+1}-frac2{r+1}+frac2{r+2}.end{align}Therefore, your series is naturally the sum of two telescoping series and its sum is$$frac12-frac23=-frac16.$$






                  share|cite|improve this answer









                  $endgroup$













                  • $begingroup$
                    (+1) was a minute late!
                    $endgroup$
                    – TheSimpliFire
                    Dec 31 '18 at 15:54
















                  7












                  $begingroup$

                  Note thatbegin{align}frac{2-r}{r(r+1)(r+2)}&=frac1r-frac3{r+1}+frac2{r+2}\&=frac1r-frac1{r+1}-frac2{r+1}+frac2{r+2}.end{align}Therefore, your series is naturally the sum of two telescoping series and its sum is$$frac12-frac23=-frac16.$$






                  share|cite|improve this answer









                  $endgroup$













                  • $begingroup$
                    (+1) was a minute late!
                    $endgroup$
                    – TheSimpliFire
                    Dec 31 '18 at 15:54














                  7












                  7








                  7





                  $begingroup$

                  Note thatbegin{align}frac{2-r}{r(r+1)(r+2)}&=frac1r-frac3{r+1}+frac2{r+2}\&=frac1r-frac1{r+1}-frac2{r+1}+frac2{r+2}.end{align}Therefore, your series is naturally the sum of two telescoping series and its sum is$$frac12-frac23=-frac16.$$






                  share|cite|improve this answer









                  $endgroup$



                  Note thatbegin{align}frac{2-r}{r(r+1)(r+2)}&=frac1r-frac3{r+1}+frac2{r+2}\&=frac1r-frac1{r+1}-frac2{r+1}+frac2{r+2}.end{align}Therefore, your series is naturally the sum of two telescoping series and its sum is$$frac12-frac23=-frac16.$$







                  share|cite|improve this answer












                  share|cite|improve this answer



                  share|cite|improve this answer










                  answered Dec 31 '18 at 15:52









                  José Carlos SantosJosé Carlos Santos

                  178k24139251




                  178k24139251












                  • $begingroup$
                    (+1) was a minute late!
                    $endgroup$
                    – TheSimpliFire
                    Dec 31 '18 at 15:54


















                  • $begingroup$
                    (+1) was a minute late!
                    $endgroup$
                    – TheSimpliFire
                    Dec 31 '18 at 15:54
















                  $begingroup$
                  (+1) was a minute late!
                  $endgroup$
                  – TheSimpliFire
                  Dec 31 '18 at 15:54




                  $begingroup$
                  (+1) was a minute late!
                  $endgroup$
                  – TheSimpliFire
                  Dec 31 '18 at 15:54











                  5












                  $begingroup$

                  Note that begin{align}sum_{r=2}^inftyleft( frac{1}{r}-frac{3}{r+1}+frac{2}{r+2}right)&=sum_{r=2}^inftyleft(frac1r-frac1{r+1}right)-2sum_{r=2}^inftyleft(frac1{r+1}-frac1{r+2}right)\&=left(frac12-frac13+frac13-frac14+cdotsright)-2left(frac13+frac14-frac14+frac15-cdotsright)\&=frac12-frac23=-frac16end{align}






                  share|cite|improve this answer









                  $endgroup$









                  • 1




                    $begingroup$
                    (+1) Believe me: I know what it feels to be a few seconds too late.
                    $endgroup$
                    – José Carlos Santos
                    Dec 31 '18 at 15:55
















                  5












                  $begingroup$

                  Note that begin{align}sum_{r=2}^inftyleft( frac{1}{r}-frac{3}{r+1}+frac{2}{r+2}right)&=sum_{r=2}^inftyleft(frac1r-frac1{r+1}right)-2sum_{r=2}^inftyleft(frac1{r+1}-frac1{r+2}right)\&=left(frac12-frac13+frac13-frac14+cdotsright)-2left(frac13+frac14-frac14+frac15-cdotsright)\&=frac12-frac23=-frac16end{align}






                  share|cite|improve this answer









                  $endgroup$









                  • 1




                    $begingroup$
                    (+1) Believe me: I know what it feels to be a few seconds too late.
                    $endgroup$
                    – José Carlos Santos
                    Dec 31 '18 at 15:55














                  5












                  5








                  5





                  $begingroup$

                  Note that begin{align}sum_{r=2}^inftyleft( frac{1}{r}-frac{3}{r+1}+frac{2}{r+2}right)&=sum_{r=2}^inftyleft(frac1r-frac1{r+1}right)-2sum_{r=2}^inftyleft(frac1{r+1}-frac1{r+2}right)\&=left(frac12-frac13+frac13-frac14+cdotsright)-2left(frac13+frac14-frac14+frac15-cdotsright)\&=frac12-frac23=-frac16end{align}






                  share|cite|improve this answer









                  $endgroup$



                  Note that begin{align}sum_{r=2}^inftyleft( frac{1}{r}-frac{3}{r+1}+frac{2}{r+2}right)&=sum_{r=2}^inftyleft(frac1r-frac1{r+1}right)-2sum_{r=2}^inftyleft(frac1{r+1}-frac1{r+2}right)\&=left(frac12-frac13+frac13-frac14+cdotsright)-2left(frac13+frac14-frac14+frac15-cdotsright)\&=frac12-frac23=-frac16end{align}







                  share|cite|improve this answer












                  share|cite|improve this answer



                  share|cite|improve this answer










                  answered Dec 31 '18 at 15:53









                  TheSimpliFireTheSimpliFire

                  13.6k62765




                  13.6k62765








                  • 1




                    $begingroup$
                    (+1) Believe me: I know what it feels to be a few seconds too late.
                    $endgroup$
                    – José Carlos Santos
                    Dec 31 '18 at 15:55














                  • 1




                    $begingroup$
                    (+1) Believe me: I know what it feels to be a few seconds too late.
                    $endgroup$
                    – José Carlos Santos
                    Dec 31 '18 at 15:55








                  1




                  1




                  $begingroup$
                  (+1) Believe me: I know what it feels to be a few seconds too late.
                  $endgroup$
                  – José Carlos Santos
                  Dec 31 '18 at 15:55




                  $begingroup$
                  (+1) Believe me: I know what it feels to be a few seconds too late.
                  $endgroup$
                  – José Carlos Santos
                  Dec 31 '18 at 15:55











                  1












                  $begingroup$

                  Actually your finite sum is equal to $$sum_{r=2}^nfrac{2-r}{r(r+1)(r+2)}=frac{2}{n+2}-frac{1}{n+1}-frac{1}{6}$$






                  share|cite|improve this answer









                  $endgroup$


















                    1












                    $begingroup$

                    Actually your finite sum is equal to $$sum_{r=2}^nfrac{2-r}{r(r+1)(r+2)}=frac{2}{n+2}-frac{1}{n+1}-frac{1}{6}$$






                    share|cite|improve this answer









                    $endgroup$
















                      1












                      1








                      1





                      $begingroup$

                      Actually your finite sum is equal to $$sum_{r=2}^nfrac{2-r}{r(r+1)(r+2)}=frac{2}{n+2}-frac{1}{n+1}-frac{1}{6}$$






                      share|cite|improve this answer









                      $endgroup$



                      Actually your finite sum is equal to $$sum_{r=2}^nfrac{2-r}{r(r+1)(r+2)}=frac{2}{n+2}-frac{1}{n+1}-frac{1}{6}$$







                      share|cite|improve this answer












                      share|cite|improve this answer



                      share|cite|improve this answer










                      answered Dec 31 '18 at 15:54









                      Dr. Sonnhard GraubnerDr. Sonnhard Graubner

                      79.9k42867




                      79.9k42867























                          1












                          $begingroup$

                          Let $dfrac{2-r}{r(r+1)(r+2)}=f(r)-f(r+1),$ where$ f(m)=dfrac{am+b}{m(m+1)}$



                          $implies2-r=(r+2)(ar+b)-r(ar+a+b)=ar+2b$



                          Set $2b=2iff b=1,ar=-riff a=-1$



                          $$impliessum_{r=1}^ndfrac{2-r}{r(r+1)(r+2)}=sum_{r=1}^nleft(f(r)-f(r+1)right)=f(1)-f(n+1)$$



                          Set $ntoinfty$ and show that $ntoinfty f(n)=0$






                          share|cite|improve this answer









                          $endgroup$


















                            1












                            $begingroup$

                            Let $dfrac{2-r}{r(r+1)(r+2)}=f(r)-f(r+1),$ where$ f(m)=dfrac{am+b}{m(m+1)}$



                            $implies2-r=(r+2)(ar+b)-r(ar+a+b)=ar+2b$



                            Set $2b=2iff b=1,ar=-riff a=-1$



                            $$impliessum_{r=1}^ndfrac{2-r}{r(r+1)(r+2)}=sum_{r=1}^nleft(f(r)-f(r+1)right)=f(1)-f(n+1)$$



                            Set $ntoinfty$ and show that $ntoinfty f(n)=0$






                            share|cite|improve this answer









                            $endgroup$
















                              1












                              1








                              1





                              $begingroup$

                              Let $dfrac{2-r}{r(r+1)(r+2)}=f(r)-f(r+1),$ where$ f(m)=dfrac{am+b}{m(m+1)}$



                              $implies2-r=(r+2)(ar+b)-r(ar+a+b)=ar+2b$



                              Set $2b=2iff b=1,ar=-riff a=-1$



                              $$impliessum_{r=1}^ndfrac{2-r}{r(r+1)(r+2)}=sum_{r=1}^nleft(f(r)-f(r+1)right)=f(1)-f(n+1)$$



                              Set $ntoinfty$ and show that $ntoinfty f(n)=0$






                              share|cite|improve this answer









                              $endgroup$



                              Let $dfrac{2-r}{r(r+1)(r+2)}=f(r)-f(r+1),$ where$ f(m)=dfrac{am+b}{m(m+1)}$



                              $implies2-r=(r+2)(ar+b)-r(ar+a+b)=ar+2b$



                              Set $2b=2iff b=1,ar=-riff a=-1$



                              $$impliessum_{r=1}^ndfrac{2-r}{r(r+1)(r+2)}=sum_{r=1}^nleft(f(r)-f(r+1)right)=f(1)-f(n+1)$$



                              Set $ntoinfty$ and show that $ntoinfty f(n)=0$







                              share|cite|improve this answer












                              share|cite|improve this answer



                              share|cite|improve this answer










                              answered Dec 31 '18 at 16:07









                              lab bhattacharjeelab bhattacharjee

                              229k15159280




                              229k15159280























                                  1












                                  $begingroup$

                                  If $H_{n}$ is the $n$-th harmonic number, we have $$sum_{r=2}^{n}left(frac{1}{r}-frac{3}{r+1}+frac{2}{r+2}right)=left(H_{n}-1right)-3left(H_{n}-1-frac{1}{2}right)+2left(H_{n}-1-frac{1}{2}-frac{1}{3}right)$$ $$=-1+frac{3}{2}-frac{2}{3}=-frac{1}{6}.$$ Now take the limit as $nrightarrow+infty$.






                                  share|cite|improve this answer









                                  $endgroup$


















                                    1












                                    $begingroup$

                                    If $H_{n}$ is the $n$-th harmonic number, we have $$sum_{r=2}^{n}left(frac{1}{r}-frac{3}{r+1}+frac{2}{r+2}right)=left(H_{n}-1right)-3left(H_{n}-1-frac{1}{2}right)+2left(H_{n}-1-frac{1}{2}-frac{1}{3}right)$$ $$=-1+frac{3}{2}-frac{2}{3}=-frac{1}{6}.$$ Now take the limit as $nrightarrow+infty$.






                                    share|cite|improve this answer









                                    $endgroup$
















                                      1












                                      1








                                      1





                                      $begingroup$

                                      If $H_{n}$ is the $n$-th harmonic number, we have $$sum_{r=2}^{n}left(frac{1}{r}-frac{3}{r+1}+frac{2}{r+2}right)=left(H_{n}-1right)-3left(H_{n}-1-frac{1}{2}right)+2left(H_{n}-1-frac{1}{2}-frac{1}{3}right)$$ $$=-1+frac{3}{2}-frac{2}{3}=-frac{1}{6}.$$ Now take the limit as $nrightarrow+infty$.






                                      share|cite|improve this answer









                                      $endgroup$



                                      If $H_{n}$ is the $n$-th harmonic number, we have $$sum_{r=2}^{n}left(frac{1}{r}-frac{3}{r+1}+frac{2}{r+2}right)=left(H_{n}-1right)-3left(H_{n}-1-frac{1}{2}right)+2left(H_{n}-1-frac{1}{2}-frac{1}{3}right)$$ $$=-1+frac{3}{2}-frac{2}{3}=-frac{1}{6}.$$ Now take the limit as $nrightarrow+infty$.







                                      share|cite|improve this answer












                                      share|cite|improve this answer



                                      share|cite|improve this answer










                                      answered Dec 31 '18 at 16:08









                                      Marco CantariniMarco Cantarini

                                      28.8k23373




                                      28.8k23373























                                          0












                                          $begingroup$

                                          By definition,



                                          $$ sum_{r=1}^infty a_r = lim_{ntoinfty}sum_{r=1}^n a_r .$$



                                          In your case the above limit is zero. Then:



                                          $$ sum_{r=2}^infty a_r = sum_{r=1}^infty a_r - a_1 = -a_1 . $$



                                          This reasoning is what @HenriLee was referring to in his answer.






                                          share|cite|improve this answer









                                          $endgroup$


















                                            0












                                            $begingroup$

                                            By definition,



                                            $$ sum_{r=1}^infty a_r = lim_{ntoinfty}sum_{r=1}^n a_r .$$



                                            In your case the above limit is zero. Then:



                                            $$ sum_{r=2}^infty a_r = sum_{r=1}^infty a_r - a_1 = -a_1 . $$



                                            This reasoning is what @HenriLee was referring to in his answer.






                                            share|cite|improve this answer









                                            $endgroup$
















                                              0












                                              0








                                              0





                                              $begingroup$

                                              By definition,



                                              $$ sum_{r=1}^infty a_r = lim_{ntoinfty}sum_{r=1}^n a_r .$$



                                              In your case the above limit is zero. Then:



                                              $$ sum_{r=2}^infty a_r = sum_{r=1}^infty a_r - a_1 = -a_1 . $$



                                              This reasoning is what @HenriLee was referring to in his answer.






                                              share|cite|improve this answer









                                              $endgroup$



                                              By definition,



                                              $$ sum_{r=1}^infty a_r = lim_{ntoinfty}sum_{r=1}^n a_r .$$



                                              In your case the above limit is zero. Then:



                                              $$ sum_{r=2}^infty a_r = sum_{r=1}^infty a_r - a_1 = -a_1 . $$



                                              This reasoning is what @HenriLee was referring to in his answer.







                                              share|cite|improve this answer












                                              share|cite|improve this answer



                                              share|cite|improve this answer










                                              answered Dec 31 '18 at 15:57









                                              Dog_69Dog_69

                                              6361523




                                              6361523






























                                                  draft saved

                                                  draft discarded




















































                                                  Thanks for contributing an answer to Mathematics Stack Exchange!


                                                  • Please be sure to answer the question. Provide details and share your research!

                                                  But avoid



                                                  • Asking for help, clarification, or responding to other answers.

                                                  • Making statements based on opinion; back them up with references or personal experience.


                                                  Use MathJax to format equations. MathJax reference.


                                                  To learn more, see our tips on writing great answers.




                                                  draft saved


                                                  draft discarded














                                                  StackExchange.ready(
                                                  function () {
                                                  StackExchange.openid.initPostLogin('.new-post-login', 'https%3a%2f%2fmath.stackexchange.com%2fquestions%2f3057821%2fevaluate-sum-r-2-infty-frac2-rrr1r2%23new-answer', 'question_page');
                                                  }
                                                  );

                                                  Post as a guest















                                                  Required, but never shown





















































                                                  Required, but never shown














                                                  Required, but never shown












                                                  Required, but never shown







                                                  Required, but never shown

































                                                  Required, but never shown














                                                  Required, but never shown












                                                  Required, but never shown







                                                  Required, but never shown







                                                  Popular posts from this blog

                                                  Bundesstraße 106

                                                  Ida-Boy-Ed-Garten

                                                  Verónica Boquete